SU(2)SU(2)SU(2) Eichsymmetrie

Nehmen Sie den Lagrange-Operator mit einem Fermion:

L = 1 4 F A μ v F μ v A + ψ ¯ ( ich γ μ D μ M ) ψ
wobei die kovariante Ableitung der Eichung D μ = μ + ich G 2 T A W μ A . Die Lagrange-Funktion ist invariant unter einem Lokal S U ( 2 ) Transformation:
ψ ( X ) exp [ ich θ A ( X ) T A ] ψ ( X )
W μ A ( X ) W μ A ( X ) + 1 G μ θ A ( X ) + ϵ A B C θ B ( X ) W μ C ( X )

Oft sagen wir das W μ A transformiert gemäß der adjungierten Darstellung von S U ( 2 ) aber wie können wir das basierend auf der vorherigen Gleichung sagen?

Antworten (1)

Beachten Sie, dass die endliche Transformation von:

W μ A W μ A + 1 G μ θ A + ϵ A B C θ B W μ C
Ist:
(1) W μ A T A G W μ A T A G 1 + ich G μ G
Wo:
G = exp ( ich θ A T A ) Und [ T A , T B ] = ich ϵ A B C T C
Also der erste Term auf der rechten Seite der Gleichung ( 1 ) transformiert sich unter die adjungierte Darstellung der Lie-Gruppe . Der zweite Term wird unter der adjungierten Darstellung nicht transformiert, aber es sollte leicht zu überprüfen sein, dass das transformierte Eichfeld immer noch Werte in der Lie-Algebra annimmt (Hinweis: Das Betrachten von infinitesimalen Transformationen ist die einfachste Methode, dies zu überprüfen).

Falls Sie weitere Informationen zur adjungierten Darstellung der Lie-Gruppe wünschen, lohnt es sich möglicherweise, sich diese Frage anzusehen .

Vielen Dank für Ihre Antwort. Ich denke, mein Problem ist, dass ich falsch verstehe, was "transformiert unter" wirklich bedeutet. Sagen Sie mir, wenn ich falsch liege: Nehmen wir S U ( 3 ) , für die 3 mit Dynkin-Indizes (1,0) transformiert sich ein Zustand wie folgt: ψ ( X ) G ψ ( X ) . Für die 3 ¯ (0,1), ein Zustand transformiert sich wie folgt: ϕ ( X ) ϕ ( X ) G 1 . Und für die adjungierte Darstellung (1,1) : Ö G Ö G 1 . Aber dann, wenn ich zurückkomme S U ( 2 ) , weil das 2 Und 2 ¯ äquivalent sind, sollten sie sich auf die gleiche Weise umwandeln?
@KoObO Um ehrlich zu sein, bin ich mit Dynkin-Indizes nicht wirklich vertraut. Ein Feld, das Werte in der Lie-Algebra annimmt, dh ϕ G , wird sich unter der adjungierten Darstellung als transformieren ϕ G ϕ G 1 und nimmt wieder Werte in der adjungierten Darstellung an, dh G ϕ G 1 G . Der einfachste Weg, dies zu verstehen, ist, sich meine hier gegebene Antwort anzusehen . Das ist der springende Punkt bei der Transformationseigenschaft des Eichfelds
sie leben in der Lie-Algebra vor und nach einer Transformation. Dies ist wahrscheinlich der Grund, warum die Leute sagen, dass sich die Eichfelder unter der adjungierten Darstellung transformieren.
Ok ich glaube ich habe verstanden. Danke für deine Antwort(en)!
@KoObO kein Problem! Wenn Sie auch die Antwort auf Ihren ersten Kommentar (in Bezug auf Dynkin-Indizes) wissen möchten, dann denke ich, dass es eine sehr berechtigte Frage ist, sie hier zu stellen. Mich persönlich würde eine Antwort interessieren!